Christina jarred 21 liters of jam after 3 days. How much jam did she jar if she spent 7 days making jam?

Answers

Answer 1

Answer:

49 liters

Explanation:

We know that Christina jarred 21 liters of jam in 3 days, so using this ratio, we can calculate the how much jam she jarred after 7 days as follows

[tex]7\text{ days}\times\frac{21\text{ liters}}{\text{ 3 days}}=49\text{ liters}[/tex]

Therefore, the answer is 49 liters.


Related Questions

Equivalent Linear Expressions MC 2.02
Which expression is equivalent to (-3 1/3d + 3/4) - (3 5/6d + 7/8)
A: 1/2d-1/8
B: 1/2d-1 5/8
C: 43/6d-1/8
D: -43/6d-1/8

Answers

The expression is equivalent to the given expression is -43/6d-1/8.

What is expression?

An expression in math is a sentence with a minimum of two numbers or variables and at least one math operation.

Given an expression [tex]-3\frac{1}{3d} +\frac{3}{4} - (3\frac{5}{6d}+\frac{7}{8} )[/tex]

= -10/3d +3/4 - 23/6d - 7/8

= -10/3d - 23/6d + 3/4 - 7/8

= -43/6d - 1/8

Hence, The expression is equivalent to the given expression is -43/6d-1/8.

For more references on expression, click;

https://brainly.com/question/16804733

#SPJ1

Answer:

- 4/15x + 4

Step-by-step explanation:

oooooooo

Which expression represents the simplest factorization of 56st – 21t?

Answers

The expression which represents the simplest factorization of 56st – 21t is 7t(8s - 3).

How to factorise an expression?

Factorization is the process of creating a list of factors. It is also an expression listing items that when multiplied together will produce a desired quantity.

Greatest common factor is the largest positive integer or polynomial that is a divisor of several different numbers.

56st - 21t

Factors of

56st = 1, 2, 4, 7, 8, 14, 28,:56, t, s

21t = 1, 3, 7, 21

The common factors of 56st and 21t is 7 and t

56st - 21t

= 7t(8s - 3)

Check:

7t(8s - 3)

= 56st - 21t

In conclusion, the simplest factorization of the expression 56st - 21t is 7t(8s - 3).

Learn more about factorization:

https://brainly.com/question/11434122

#SPJ1

Not homework just review for test not worth any points

Answers

Recall that:

Interest is the monetary gain for lending money to a third party.

Simple interest is applying the interest to the original amount without considering the extra flow of money each time the interest is applied.

Compounded interest is applying the interest to the total amount including the extra flow of money each time the interest is applied.

1) Plan A. From the given information, in 12 months Ari will earn:

[tex]I=500\cdot0.065\cdot12=390.[/tex]

dollars, therefore she will have a total of $890.00.

2) Plan B. Now we use the formula for compounded interest to compute the total amount that Ari will have after one year:

[tex]A=500(1+0.065)^{12}=500\cdot2.129096\approx1064.55.[/tex]

3) From the above calculations, we conclude that Ari will make more money if she invests in plan B.

4) Ari has to leave the money for another:

Using plan B. Let t be the number of extra months Ari needs to leave the money in the fund, then:

[tex]1100=500\cdot(1.065)^{12+t}\text{.}[/tex]

Solving for t, we get:

[tex]\begin{gathered} 1.065^{12+t}=2.2, \\ (12+t)ln(1.065)=\ln (2.2), \\ 12+t=\frac{\ln (2.2)}{\ln (1.065)}=12.52, \\ t=12.52-12=0.52. \end{gathered}[/tex]

Then, Ari needs to leave the money for another half a month but since the interest is applied each month she needs to leave the money for another whole month.

Using Plan A: Let t be the number of extra months Ari needs to leave the money in the fund, then:

[tex]1100=500\cdot0.065\cdot(12+t)\text{ +500.}[/tex]

Solving for t we get:

[tex]\begin{gathered} \frac{1100-500}{500\cdot0.065}=12+t, \\ t=\frac{600}{32.5}-12, \\ t=6.46. \end{gathered}[/tex]

Then, Ari needs to leave the money for another 6.46 months but since the interest is applied monthly she needs to leave the money for another 7 months.

What is (6/7)/(2/7)?

Answers

To find the value of;

[tex]\frac{(\frac{6}{7})}{(\frac{2}{7})}[/tex]

When dividing fractions, for example a divided by b is equals a times 1/b;

[tex]a\text{ divided by b = a }\times\frac{1}{b}[/tex]

note that the divisor which is b is inversed and multiplied.

So, let us apply the same rule to the given question.

The divisor which is the second fraction for the question is 2/7, we need to inverse 2/7 and multiply it by the first fraction.

The inverse of 2/7 is 7/2, So;

[tex]\frac{(\frac{6}{7})}{(\frac{2}{7})}=\text{ }\frac{6}{7}\times\frac{7}{2}=\frac{(6\times7)}{(7\times2)}=\frac{42}{14}[/tex]

And finally;

[tex]\frac{42}{14}=3[/tex]

Therefore the final answer is 3.

[tex]\frac{(\frac{6}{7})}{(\frac{2}{7})}=\text{ 3}[/tex]

Write an equation for each scenario, then find the solution.5. You're making a Wawa run for snacks and energy drinks on your bike. It takes you0.4 hours at x miles per hour to make it to Wawa. On your way back you have toride 3 miles per hour slower (you're weighed down by your Wawa goodies) and ittakes you 0.5 hours. How far is it from your house to Wawa?6. Laura retired from her job recently, and she has saved about $500,000 over the

Answers

Let y represent the distance from your house to wawa. This means that the distance from wawa to your house is also y miles.

It takes you 0.4 hours at x miles per hour to make it to Wawa.

Distance = speed * time

Therefore, on your way to wawa, the expression would be

y = 0.4 * x = 0.4x

On your way back you have to ride 3 miles per hour slower. This means that your speed was (x - 3) miles per hour. Since it took you 0.5 hours, the expression for distance would be

y = 0.5(x - 3)

Since the distance is the same, it means that

0.4x = 0.5(x - 3)

0.4x = 0.5x - 1.5

0.5x - 0.4x = 1.5

0.1x = 1.5

x = 1.5/0.1

x = 15

Therefore,

y = 0.4x = 0.4 * 15

y = 6

The distance from your house to wawa is 6 miles

pan made a scale drawing of a swimming pool. The diving board is 2 inches long in the drawing. The actual diving board is 18 feet long. What scale did Pam use for the drawing?

Answers

Divide the length of the object on the drawing by its actual size to find the scale:

[tex]\frac{2\text{ inches}}{18\text{ feet}}[/tex]

Since each feet has 12 inches:

[tex]\frac{2\text{ inches}}{18\text{ feet}}=\frac{2\text{ inches}}{18\cdot12\text{ inches}}=\frac{2}{216}=\frac{1}{108}[/tex]

Therefore, the scale used in the drawing is 1:108 (one inch in the paper equals 108 inches in real life).

Fourteen more than 4 times a number is 42what is equation?what is solution?

Answers

We have to express this phrase in mathematical terms.

We call the number x.

Then, four times a number is "4x".

Fourtenn more than 4x is "4x+14", and this is equal to 42.

Then, we can write:

[tex]4x+14=42[/tex]

And solve as:

[tex]\begin{gathered} 4x+14=42 \\ 4x=42-14 \\ 4x=28 \\ x=\frac{28}{4} \\ x=7 \end{gathered}[/tex]

Answer: the equation is 4x+14=42 and the solution is x=7

A full one- gallon container can beused to fill the one-liter containers, as shownbelow. Write a unit rate that estimates thenumber of liters per gallons.

Answers

[tex]\text{ 3 liters per gallon}[/tex]

Explanation

Step 1

as we can see in the picture

[tex]1\text{ gallons}\rightarrow3\text{ liters}[/tex]

so, to find the unit rate make

[tex]\begin{gathered} \text{unit rate=}\frac{Number\text{ of liters}}{Number\text{ of gallons}} \\ \text{replace} \\ \text{Unit rate=}\frac{3\text{ liters}}{1\text{ gallon}}=\text{ 3 liters per gallon} \end{gathered}[/tex]

I hope this helps you

7. The manager of a local restaurant has found that his cost function for producing coffee is C(x) = .097x, where C(x) is the total cost in dollars of producing x cups. (He is ignoring the cost of the coffeepot and the cost labor.) Find the total cost of producing the following numbers of cups of coffee. (a) 1000 cups (b) 1001 cups (c) What is the marginal cost for any cup? Let C(x) be the total cost in dollars to manufacture x items. Find the average cost in exercises 8 and 9.

Answers

To solve for the total cost of producing the following numbers of cups of coffee:

[tex]\begin{gathered} C(x)=0.097x \\ \end{gathered}[/tex]

where

[tex]\begin{gathered} x=nu\text{mber of cups of coff}e \\ C(x)=total\text{ cost }in\text{ dollars of producing x cups} \end{gathered}[/tex]

(a) The total cost of producing 1000 cups =

[tex]\begin{gathered} C(x)=0.097x \\ x=1000 \\ C(x)=1000(0.097)=\text{ \$97} \end{gathered}[/tex]

(b) The total cost of producing 1001 cups =

[tex]\begin{gathered} C(x)=0.097x \\ x=1001 \\ C(x)=1001(0.097)=\text{ \$97}.097 \end{gathered}[/tex]

(c) The marginal cost for any cup = $0.097

marginal cost can be found by taking the derivative of the function

[tex]\begin{gathered} C(x)=0.097x \\ C^1(x)=0.097=\text{ \$0.097} \end{gathered}[/tex]

Angel is at a birthday party. There are 12 cupcakes where some are blue and some are green. Out of There are 8 green cupcakes. Express the number of blue cupcakes as a decimal

Answers

12 cupcakes

8 green cupcakes

To find the number of blue cupcakes subtract the green cupcakes (8) to the total cupcakes:

12 -8 = 4 blue cupcakes

To express as a decimal, divide the number of blue cupcakes by the total number of cupcakes:

4/12 = 0.33333

What is the solution to the inequality x(x – 3) > 0?Question 8 options:A) x ≤ 0 or x ≥ 3B) x < 0 or x > 3C) x ≤ 0 and x ≥ 3D) 0 < x < 3

Answers

,Inequalities

It's given the inequality:

x(x - 3) > 0

Note the left side is the product of two expressions: x and x-3.

That product must be greater than zero, or positive.

Recall that the product of two numbers is positive in two different cases:

Both are positive, for example 5*4=20

Both are negative, for example (-5)*(-4) = 20

This means that we can provide two different answers to the inequality, both valid:

1. When x >0 AND x - 3 >0 (both positive), or

2. When x <0 AND x - 3 < 0 (both negative).

The first condition leads to:

x>0 AND x>3. If we intersect these conditions, the solution for this part is x>3.

The second condition gives us:

x<0 AND x<3. The intersection of these conditions gives x<0.

Thus, the final solution is the union (OR) both partial intervals above, i.e.

x>3 OR x<0 --> Option B)

Answer:

X < 0 and x > 3.

Step-by-step explanation:

chatgpt.

Find the missing side. Round to the nearest tenth.
1)
X
1.
2.
21°
16
2)
12
40°

Answers

The sides of a triangle can be determined if the trigonometric ratios and angles are given. The missing sides of the given triangle are 6 and 8 respectively.

What are trigonometric ratios?

The trigonometric ratios are defined for a right angled triangle.

The example of these ratios are sin, cos, tan, cosec, sec and cot.

The trigonometric ratios are useful in determining the heights and distances of the large objects.

The diagram for the given triangle is given below,

Since, sin(x) = (The side opposite to angle x) / Hypotenuse

Apply the above trigonometric ratio in triangle 1 to get,

Sin(21°) = x / 16

=> x = 16 × Sin(21°)

=> x = 16 × 0.36

=> x = 5.76

Which is approximately 6 when taken as nearest to tenth.

Similarly for triangle 2, it can be written as,

Sin(40°) = x / 12

=> x = 12 × Sin(40°)

=> x = 12 × 0.64

=> x = 7.68

Which is approximately 8 when taken as nearest to tenth.

Hence, the missing sides for the given triangles are 6 and 8 approximately.

To know more about trigonometric ratio click on,

https://brainly.com/question/25122825

#SPJ1

Solve the equation involving absolute value. Type you answers from smallest to largest. If an answer is not an integer then type it as a decimal rounded to the nearest hundredth. |x^2+2x-36|=12 x=Answer, Answer, Answer and Answer

Answers

Given:

The euqation is,

[tex]|x^2+2x-36|=12[/tex]

Explanation:

Simplify the equation.

[tex]\begin{gathered} |x^2+2x-36|=12 \\ x^2+2x-36=\pm12 \end{gathered}[/tex]

Solve the x^2 + 2x - 36 = 12 for x.

[tex]\begin{gathered} x^2+2x-36=12 \\ x^2+2x-36-12=0 \\ x^2+8x-6x-48=0 \\ x(x+8)-6(x-8)=0 \\ (x+8)(x-6)=0 \\ x=-8,6 \end{gathered}[/tex]

Solve the equation x^2 + 2x - 36 = -12 for x.

[tex]\begin{gathered} x^2+2x-36+12=0 \\ x^2+2x-24=0 \\ x^2+6x-4x-24=0 \\ x(x+6)-4(x+6)=0 \\ (x+6)(x-4)=0 \\ x=-6,4 \end{gathered}[/tex]

Thus solution of the equation is x = -8, -6, 4, and 6

Find the maximum value of the objective function and the values of x and y for which it occurs. F= 2x+y3x+5y ≤45 x ≥0 and y ≥02x+4y ≤32The maximum value of the objective function is ______.

Answers

The objective function is:

[tex]F=2x+y[/tex]

We need to find the shaded region where 4 inequalities overlap, then we need to graph the given inequalities:

[tex]\begin{gathered} 3x+5y\leq45 \\ 5y\leq-3x+45 \\ y\leq\frac{-3}{5}x+\frac{45}{5} \\ y\leq-\frac{3}{5}x+9 \end{gathered}[/tex]

And the other one:

[tex]\begin{gathered} 2x+4y\leq32 \\ 4y\leq-2x+32 \\ y\leq\frac{-2x}{4}+\frac{32}{4} \\ y\leq-\frac{1}{2}x+8 \end{gathered}[/tex]

And x>=0, y>=0

The graph is:

The coordinates of the shaded region are then:

(0,0), (0,8), (10,3) and (15,0)

To obtain the maximum value, let's evaluate the objective function in all the coordinates:

[tex]\begin{gathered} F(0,0)=2\times0+0=0+0=0 \\ F(0,8)=2\times0+8=0+8=8 \\ F(10,3)=2\times10+3=20+3=23 \\ F(15,0)=2\times15+0=30+0=30 \end{gathered}[/tex]

Then, the maximum value is the largest value obtained, then it's 30 and it occurs at x=15 and y=0.

Find the length of x.43a. 2.5b. 3C. 4d. 5e. 656

Answers

the given triangle is a right angle triangle.

by Pythagoras theorem.

4^2 + 3^2 = x^2

16 + 9 = X^2

25 = x^2

[tex]\begin{gathered} x^2=25 \\ x=5 \end{gathered}[/tex]

so, the answer is x = 5.

nine cubes are glued together to form the solid shown in the digram .

Answers

ANSWER

D. 272.25 mm²

EXPLANATION

First we have to find the area of one face of one cube:

[tex]A=2.75^2=7.5625\operatorname{mm}^2[/tex]

Now we have to count how many faces of the squares are in the surface of the solid. Or a faster way is to multiply the total number of faces of the 9 cubes: 6*9=54 and subtract the number of faces that are not on the surface:

There are 18 faces of the cubes that are not on the surface of the solid. The number of faces from the 9 cubes that are on the surface of the solid is:

[tex]54-18=36[/tex]

The surface area of the solid is the area of a face of one cube multiplied by the number of faces on the surface of the solid:

[tex]SA=36\cdot A=36\cdot7.5625=272.25\operatorname{mm}^2[/tex]

Answer:

D. 272.25 mm²

Step-by-step explanation:

11. Determine if the following sequence is arithmetic or geometric. Then, find the 12th term. 2, 6, 18, 54, ... a. arithmetic: 35 b. arithmetic: 354,294 c. geometric: 35 d. geometric: 354,294

Answers

We have the sequence: 2, 6, 18, 54...

If the sequence is arithmetic, there must be a common difference between the terms that remains constant.

This is not the case for this sequence.

We can try by seeing if there is a common factor k such that:

[tex]a_n=k\cdot a_{n-1}[/tex]

We can do it by:

[tex]\frac{a_2}{a_1}=\frac{6}{2}=3[/tex][tex]\frac{a_3}{a_2}=\frac{18}{6}=3[/tex][tex]\frac{a_4}{a_3}=\frac{54}{18}=3[/tex]

There, we have a geometric sequence, with factor k=3:

[tex]a_n=3\cdot a_{n-1}[/tex]

We can relate it to the first term as:

[tex]\begin{gathered} a_2=3\cdot a_1 \\ a_3=3\cdot a_2=3\cdot3\cdot a_1=3^2\cdot a_1 \\ a_4=3\cdot a_3=3\cdot3^2\cdot a_1=3^3\cdot a_1 \\ a_n=3^{n-1}\cdot a_1=3^{n-1}_{}\cdot2 \end{gathered}[/tex]

For n=12, we have:

[tex]a_{12}=3^{12-1}\cdot2=3^{11}\cdot2=177,147\cdot2=354,294[/tex]

The value of a12 is 354,294.

The answer is d) Geometric, 354,294.

Find the following values of the function6(-7+8f(x) = 75 - 2.c12-2 < -2– 2 4f(-4)=f(-2) =f(-1) =f(9) =91)Answers in progress)

Answers

Find the following values of the function

6

(-7+8

f(x) = 7

5 - 2.c

12

-

2 < -2

– 2a> 4

f(-4)=

f(-2) =

f(-1) =

f(9) =

9

1)

I will show you the pic

Answers

If it rotates clockwise around the origin. It will be in the following quadrant

[tex]\begin{gathered} 90^{\circ}\rightarrow quadrant\text{ 1} \\ 180^{\circ}\rightarrow quadrant\text{ 4} \\ 270^{\circ}\rightarrow quadrant\text{ 3} \\ 360^{\circ^{\prime}^{\prime}}\rightarrow quadrant\text{ 2} \end{gathered}[/tex]

Solve by using substitution.=−3−3=−4−5

Answers

hdhduxhx, this is the solution to the exercise:

Multiplying by -1 the second equation, we have:

y = -3x - 3

-y = 4x + 5

____________

0 = x + 2

x = -2

_____________

Now we can solve for y in the first equation, this way:

y = - 3x - 3

y = -3 (-2) - 3

y = 6 - 3

y = 3

____________

Finally, let's prove that x = -2 and y = 3 is correct for the second equation, as follows:

y = -4x - 5

3 = -4 (-2) - 5

3 = 8 - 5

3 = 3

___________

We proved that x = -2 and y = 3 is correct.

The following data set represents the ACT scores for students in Mrs. Smith's collegescience class.18 32 16 2319 21 28 2922 30 19 21What ACT score is the mode for the class?

Answers

Recall that in statistics, the mode is the value that appears most frequently in a set of data. You may find it useful to represent the data in statistical graphs such as dot plots or bar charts. In this case, we will use the following chart

As we can see from the image, The most frequently used numbers are 19 and 21. We are dealing with a bimodal data set. The mode is given by 19 and 21.

Find f in:f ÷ -2/3 = -1/3

Answers

ANSWER

[tex]f\text{ = }\frac{2}{9}[/tex]

EXPLANATION

We want to find f in:

[tex]f\text{ }\div\text{ -}\frac{2}{3}\text{ = -}\frac{1}{3}[/tex]

To do that, we multiply both sides by -2/3.

That way, it cancels out the -2/3 on the left hand side and isolates f.

So we have:

[tex]\begin{gathered} f\text{ }\div(-\frac{2}{3})\cdot\text{ (-}\frac{2}{3})\text{ = -}\frac{1}{3}\cdot\text{ -}\frac{2}{3} \\ f\text{ = }\frac{2}{9} \end{gathered}[/tex]

That is the answer.

in the diagram pq and St find the slope of st

Answers

[tex]m_{ST}=-1[/tex]

1) Looking at that diagram, we can see that Point S shares the same x-coordinate from point Q since they are on the same Vertical Line. So, it's safe to say that the x-coordinate of Point S is x=-2

2) And point S is at the same horizontal axis of point P. So for the same principle, we can say that the y-coordinate of point S is y=4

S(-2,4)

3) So let's find the Fourth point of that figure, so let's find the slope of PQ

[tex]\begin{gathered} m_{PQ}=\frac{4-1}{1-(-2)}=\frac{3}{-1+2}=\frac{3}{3}=1 \\ \\ m_{ST}=-1 \end{gathered}[/tex]

Since PQ is perpendicular to ST then the slope is the opposite reciprocal of 1, i.e. -1

I really don't understand this lesson can you help me please

Answers

Data:

M = Math books

E = Englisg books

TB = Total books

The total of the books is the addition of the math and english books

[tex]TB\text{ = M+ E}[/tex]

The Math books are: M= E-9 (Nine less that English books (E))

So replacing this in the expression for Total Books:

[tex]TB=(E-9)+E\text{ }\Rightarrow TB=E-9+E[/tex]The final expression is:

[tex]TB=2E-9[/tex]

Look at the models below. DO Tell whether each statement is True or False. equivalent to į because à has 2 times as many shaded parts and 2 times as many equal parts as True False 3 b. , is equivalent to because has 2 times as many parts shaded and 2 more equal parts than True 0 False 4 is equivalent to because both models have i shaded part True False d. is equivalent to because á has 2 + 6 = 8 equal parts. True False

Answers

a.

Since:

[tex]\frac{1}{3}\equiv\frac{2}{2}\times\frac{1}{3}=\frac{2}{6}[/tex]

The statement is true

b.

Since:

[tex]\frac{2}{2}\times\frac{1}{4}=\frac{2}{8}\ne\frac{2}{6}[/tex]

The statement is false

c.

Since:

[tex]\frac{1}{4}\equiv\frac{2}{2}\times\frac{1}{4}=\frac{2}{8}[/tex]

The statement is true.

d.

Since:

[tex]\begin{gathered} \frac{2}{6}=\frac{1}{3} \\ \frac{2}{8}=\frac{1}{4} \\ \frac{1}{4}\ne\frac{1}{3} \end{gathered}[/tex]

The statement is false.

What is the connection between side ratios and angle in a right triangle

Answers

Trigonometric ratios are ratios between any two sides of a right triangle that can then be used to determine the measure of an angle between those two sides. Primary trigonometric ratios (in a right triangle trigonometry) are: sin (x), cos (x) and tan (x)

On the basis of any two side lengths, we may even determine the acute angle measurements in a right triangle. We may get the ratios of the lengths of the triangle's sides to an acute angle in a right triangle by knowing the acute angle's measurement.

What relationship exists between a triangle's side ratios and angle?

In a triangle, the longest side is located across from the largest angle, while the shortest side is located across from the smallest angle.

Triangle Unfairness: The lengths of any two sides added together in a triangle always exceed the length of the third side.

Pythagoras Theorem, a²+b²=c² in a right triangle with hypotenuse c.

A triangle is a right triangle if its sides conform to the relationship a²+b²=c²

The right angle, or 90°, is always one angle. The hypotenuse is the side with the 90° angle opposite. The longest side is always the hypotenuse. The other two inner angles add up to 90 degrees.

Remember that the Pythagorean theorem only holds true for right triangles, in which case the converse of the theorem also holds true. In other words, if a triangle's sides satisfy the connection, then it must be a right triangle.

To know more about the Pythagorean theorem, visit:

https://brainly.com/question/14930619

#SPJ1

Instructions: varies indirectly with . If =7 when =−4, find when =2. Use the forward slash (i.e. "/") for all fractions (e.g. -1/2 is the same as −12).

Answers

Answer:[tex]\begin{gathered} Equation:\text{ y=}\frac{-7}{4}x \\ \\ Solution:\text{ y=}\frac{-7}{2} \end{gathered}[/tex]

Explanation:

y varies directly as x

This can be written mathematically as:

y = kx

If y = 7, x = -4

Solve for the constant k

7 = -4k

k = -7/4

The equation is:

y = -7/4 x

When x = 2

substitute x =2 anto the equatio n y = -7/4 x

[tex]\begin{gathered} y=-\frac{7}{4}\times2 \\ \\ y=-\frac{7}{2} \end{gathered}[/tex]

3. The difference of two-thirds of a number x and6 is at least -24. Which inequality represents allpossible values for x?

Answers

We will solve as follows:

The expression represented in the text is:

[tex]\frac{2}{3}x-6\ge-24[/tex]

Now, we solve for x:

[tex]\Rightarrow\frac{2}{3}x\ge-18\Rightarrow2x\ge-54[/tex][tex]\Rightarrow x\ge-27[/tex]

So, the solution is x >= 27. [Option D]

Write an equation in slope-intercept form for the line that is parallel to y = 3x + 7 and passes through the point (-4, -8).

Answers

The line parallel to y = 3x +7 and passes through the point (-4,-8) is

y=3x+4

What is a slope-intercept form?

It gives the graph of a straight line and it is represented in the form

y= mx +c. It is one of the form used to calculate the equation of a straight line. We have to calculate the slope of the line from the equation. The slope calculated can be used in the slope-intercept form. It is the most popular form of a straight line.

We need to find the slope to the line y = 3x +7.

The slope of a line is m

Here, from the equation y=3x+7, m=3

So, m=3

The slope-intercept form is,

y= mx +c

-8 = 3(-4)+c

-8=-12+c

c=4 and m=3

Now, the above equation equation becomes,

y=3x+4

The line parallel to y = 3x +7 and passes through the point (-4,-8) is

y=3x+4

To know more about slope-intercept form, visit:

https://brainly.com/question/9682526

#SPJ1

Consider the functions g(x) = 2x + 1 and h(x) = 2x + 2 for the domain 0 < x < 5 Without evaluating or graphing the functions, how do the ranges compare?

Answers

Without evaluating or graphing the functions, it is obvious that the minimum and maximum values in the range of h(x) is greater than the minimum and maximum values in the range of g(x) by 1.

What is a domain?

In Mathematics, a domain simply refers to the set of all real numbers for which a particular function is defined. This ultimately implies that, a domain represents the input values (x-values) to a function.

What is a range?

In Mathematics, a range can be defined as the set of all real numbers that connects with the elements of a domain. This ultimately implies that, a range refers to the set of all possible output numerical values (real numbers), which are shown on the y-axis of a graph.

In conclusion, the minimum and maximum values in the range of h(x) differs from those of function g(x) by 1.

Read more on range here: https://brainly.com/question/12077664

#SPJ1

Other Questions
when you compare the 2016-2017 season with the 2017-2018 season, what was the percent increase in the number of games that the Lakers won ? show your work. f(x) = log x + 2 and g(x) = log (1/x). Find (f g) (x).log x -2 log (1/x)22 log x + 2(2/log x) + 1 choose correct word name for the number below. 51,104 For each equation state the number of complex roots, the possible number of positive real roots,and the possible rational roots x^4+8x^2+2=0 balloon 670 meters away angle 42degrees the higher balloon is 945 away angle 36 degrees how much higher is the balloon on the right than the left Use the graph of the function F shown here to find f(1), f(2), f(3). Simplify the fraction: 1890 Is (2,-2) a solution to the system of equations.9x = 10 - 4y = 3 - 8- Meredith did some research on the ages of the male U.S. Olympic swimmers. She made a dot plot for the data . 20 21 22 23 24 25 26 27 28 29 Age of male US. Olymple swimmers years) According to Meredith's data, what is the typical age of a male U.S. Olympic swimmer? O A. 21 years O B. 26 years O C. 20 years OD. 29 years PROBLEMA 3 Con dos camiones cuyas capacidades de carga son respectivamente de 3 y 4 toneladas, se hicieron en total 23 viajes para transportar 80 toneladas de madera. Cuantos viajes realiz cada camin? what is the surface area of a rectangular prism if the measures are 13, 9, 4 The Round Up carnival ride below has a radius of 3.90 meters and rotates 0.527 times per second. As shown, riders can be held up by only friction. What coefficient of friction is needed to keep the riders from sliding down? Include units in your answer. Answer must be in 3 significant digits. Identify whether the set of ordered pairs represent an exponential. Explain your answer. x 2 0 2 4 y 4 12 36 108A. exponential functionAs the x-values are increased by a constant amount, the y-values increase by the same amount.B. not an exponential functionAs the x-values are increased by a constant amount, the y-values increase by the same amount.C. not an exponential functionAs the x-values are increased by a constant amount, the y-values are not multiplied by a constant amount.D. exponential functionAs the x-values are increased by a constant amount, the y-values are multiplied by a constant amount. ASAP PLEASEMary buy a 5 kg jar of sweets for 25. Then Mary divide the sweets into 125 g packets and sell them for 99p each.a. How many packets can Mary make?b. How much profit will Mary make? Write it Step by step. For what value of x does 3x _ gx-4? (3x+30) line p and q are parallel solve for x A psychologist is interested in constructing a 90% confidence interval for the proportion ofpeople who accept the theory that a person's spirit is no more than the complicated network ofneurons in the brain. Of those randomly selected, 50 of the 722 people agreed with this theory.a. With 90% confidence the proportion of all people who accept the theory that a person's spiritis no more than the complicated network of neurons in the brain is between ____ and ____.Round to 3 decimal places.b. If many groups of 722 randomly selected people are surveyed, then a different confidenceinterval would be produced from each group. About ___ percent of these confidenceintervals will contain the true population proportion of all people who accept the theory that aperson's spirit is no more than the complicated network of neurons in the brain and about ____percent will not contain the true population proportion. J and K are independent events. P(J | K) = 0.43. Find P(J)P(J) = I need to know what goes in the boxes for this practice question. Which example best describes a computer model that could be used to betterunderstand the function of the brain?A. A simulation predicting the paths of electrical impulses throughthe brainB. A detailed re-creation of the brain in modeling clay that shows thepositions of its structuresC. A description of the brain as a computer processor that interpretsencoded informationD. A formula for calculating the time it takes a message from thebrain to reach a different part of the body